Taylor Series Problem - Question and my attempt so far

Click For Summary
The discussion revolves around a Taylor series problem where the user seeks help with their attempt at solving it. They express uncertainty about their notes and the correctness of their Taylor expansion. A response clarifies the formula for the Taylor expansion, emphasizing the importance of evaluating the function and its derivatives at the specified point. The user is encouraged to continue from their last step and verify their calculations. Overall, the thread highlights the challenges of understanding Taylor series and the need for clear notes in mathematical problem-solving.
jammyloller
Messages
1
Reaction score
0
Question:

http://i.imgur.com/GsjeL.png

Here is my attempt so far:

http://i.imgur.com/AyOCm.png

Note: I've used m where the question has used j.

My attempt is based off some bad notes I took in class so the way I am trying to solve the problem may not be the best. I'm struggling to work out how to continue from the part where I've left a question mark, and I'm not even sure if the Taylor expansion is correct.

Could anybody offer some thoughts as to what to do?
 
Physics news on Phys.org
jammyloller said:
Question:

http://i.imgur.com/GsjeL.png

Here is my attempt so far:

http://i.imgur.com/AyOCm.png

Note: I've used m where the question has used j.

My attempt is based off some bad notes I took in class so the way I am trying to solve the problem may not be the best. I'm struggling to work out how to continue from the part where I've left a question mark, and I'm not even sure if the Taylor expansion is correct.

Could anybody offer some thoughts as to what to do?
Hello jammyloller. Welcome to PF !

attachment.php?attachmentid=54821&stc=1&d=1358466852.png


attachment.php?attachmentid=54817&stc=1&d=1358463969.jpg


I'm not sure what your notes are supposed to say, but the Taylor expansion of f(x) about the point x0 is given by
\displaystyle f(x)=f(x_0)+f&#039;(x_0)\Delta x<br /> +\frac{f&#039;&#039;(x_0)}{2!}(\Delta x)^2+\frac{f&#039;&#039;&#039;(x_0)}{3!}(\Delta x)^3+\frac{f&#039;&#039;&#039;&#039;(x_0)}{4!}(\Delta x)^4+\dots\ ,\ \ \text{ where }\ \ \Delta x=x-x_0\ .​
 

Attachments

  • AyOCm.jpg
    AyOCm.jpg
    28.6 KB · Views: 613
  • GsjeL.png
    GsjeL.png
    16.1 KB · Views: 575
Question: A clock's minute hand has length 4 and its hour hand has length 3. What is the distance between the tips at the moment when it is increasing most rapidly?(Putnam Exam Question) Answer: Making assumption that both the hands moves at constant angular velocities, the answer is ## \sqrt{7} .## But don't you think this assumption is somewhat doubtful and wrong?

Similar threads

Replies
2
Views
3K
  • · Replies 3 ·
Replies
3
Views
2K
  • · Replies 2 ·
Replies
2
Views
1K
Replies
1
Views
4K
Replies
3
Views
2K
  • · Replies 4 ·
Replies
4
Views
1K
  • · Replies 5 ·
Replies
5
Views
2K
  • · Replies 6 ·
Replies
6
Views
11K
  • · Replies 14 ·
Replies
14
Views
3K
  • · Replies 2 ·
Replies
2
Views
2K